- Wed Jan 22, 2020 4:47 pm
#73471
Complete Question Explanation
(The complete setup for this game can be found here: https://forum.powerscore.com/lsat/viewtopic.php?t=3901)
The correct answer choice is (D)
If Q performs at 12, then P must perform at 6. In addition, from the actions of the last rule, a folk group must play at 10, and from the third rule we can then deduce that two folk groups play at 10, leading to the following setup:
G, H, L, and T are the remaining groups that must still perform. From the second rule, G must perform earlier than H, and so G performs at 8 and H performs at 10. In order to conform to the third rule, L, the only other folk group, must then also perform at 10, and T is left to perform at 8. These inferences lead to the following setup:
Thus, although we cannot determine the stages that G, H, L, and T perform on, we can determine the times that they perform.
Answer choice (A) is incorrect because P must perform first on the north stage.
Answer choice (B) is incorrect because H cannot perform at 8, and T cannot perform at 10.
Answer choice (C) is incorrect because G cannot perform at 10.
Answer choice (E) is incorrect because from the fourth rule L and T cannot perform on the same stage.
Answer choice (D) is thus proven correct.
(The complete setup for this game can be found here: https://forum.powerscore.com/lsat/viewtopic.php?t=3901)
The correct answer choice is (D)
If Q performs at 12, then P must perform at 6. In addition, from the actions of the last rule, a folk group must play at 10, and from the third rule we can then deduce that two folk groups play at 10, leading to the following setup:
G, H, L, and T are the remaining groups that must still perform. From the second rule, G must perform earlier than H, and so G performs at 8 and H performs at 10. In order to conform to the third rule, L, the only other folk group, must then also perform at 10, and T is left to perform at 8. These inferences lead to the following setup:
Thus, although we cannot determine the stages that G, H, L, and T perform on, we can determine the times that they perform.
Answer choice (A) is incorrect because P must perform first on the north stage.
Answer choice (B) is incorrect because H cannot perform at 8, and T cannot perform at 10.
Answer choice (C) is incorrect because G cannot perform at 10.
Answer choice (E) is incorrect because from the fourth rule L and T cannot perform on the same stage.
Answer choice (D) is thus proven correct.
You do not have the required permissions to view the files attached to this post.